LSAT and Law School Admissions Forum

Get expert LSAT preparation and law school admissions advice from PowerScore Test Preparation.

 Administrator
PowerScore Staff
  • PowerScore Staff
  • Posts: 8917
  • Joined: Feb 02, 2011
|
#40919
Complete Question Explanation
(The complete setup for this game can be found here: lsat/viewtopic.php?t=1007)

The correct answer choice is (A)

The question stem requires you to select a pair of courses that allow for either S3 or S9 to be selected. S3 is a random, and thus no rule addresses S3. S9, on the other hand, figures within two rules:
  • The third rule establishes that S9 and W cannot be selected together. Thus, any answer choice that contains W must be eliminated. This removes answer choices (C) and (E) from further consideration.

    The fourth rule establishes that if Alicia takes P, then she must take S9. Therefore, the selection of P specifically requires S9 (which, from the condition in the game scenario eliminates her ability to take S3). Thus, selecting P violates the condition of the question stem, and any answer choice that contains P must be eliminated. This removes answer choices (B) and (D) from further consideration.
With answer choices (B), (C), (D), and (E) eliminated, answer choice (A) is proven correct by process of elimination. Note that the two variables in answer choice (A) immediately satisfy the requirement of the first and fifth rules, the two rules that caused space to be reserved within Alicia’s course list.

Get the most out of your LSAT Prep Plus subscription.

Analyze and track your performance with our Testing and Analytics Package.